mshinners
Thanks Received: 135
Atticus Finch
Atticus Finch
 
Posts: 367
Joined: March 17th, 2014
Location: New York City
 
This post thanked 1 time.
 
 

Re: Q22 - In a study, one group

by mshinners Fri Dec 31, 1999 8:00 pm

Question Type:
Weaken

Stimulus Breakdown:
Conclusion: High Protein/Low Carb diet is the best way to lose fat.
Premise: A study where we fed people different diets and measured their weight loss.

Answer Anticipation:
This question deals with one of the most common term shifts on the LSAT - weight loss vs. fat loss. The study tells us about weight loss, and the conclusion speaks to fat loss. For these questions, the LSAT almost always brings up something other than fat that would result in weight gain/loss (muscle; water weight; etc...).

It's also important to note that our study seems rather representative, and the two groups seem to have started from the same place. While a study suggests a sampling flaw, this study seems to have covered its bases.

Correct Answer:
(A)

Answer Choice Analysis:
(A) Bingo. The group that lost less weight did so because of retained water. This answer suggests the fat loss might have been comparable despite the weight loss being different.

(B) Too weak/out of scope. "Many people" isn't really enough here - a few exceptions wouldn't disprove a general trend. Also, the argument is about a high protein/low carb diet, and this answer is just about a high protein diet - what about the carbs?

(C) If anything, opposite. This answer suggests that the fat loss for this group would be even bigger than the weight loss would suggest, which would help the argument. Also, this answer doesn't compare the two diets, and, generally, answers that strengthen/weaken comparative arguments such as this one will tell you something about both "sides" of the comparison.

(D) If anything, opposite. We'd need to assume that exercise leads to weight/fat loss, but let's grant that for a second. In that case, the group that should have lost more weight/fat all things equal (low protein/high carb/exercise) lost less weight than the other group (high protein/low carb/no exercise). This would lend evidence to the argument that the high protein/low carb diet is really good for losing weight.

(E) Out of scope. The argument is about what happens while on the diet, not what happens after the diet.

Takeaway/Pattern: A study/survey generally suggests a Sampling flaw. This argument is a great example of what an argument will say when it's not committing a Sampling flaw.

When an argument is comparing things, the correct answer to a Strengthen/Weaken question will generally give you information allowing you to draw a comparison between those two things on a certain metric.

If the LSAT is talking about weight/fat loss, make sure it doesn't jump between them, as it's a hard detail creep to spot.

#officialexplanation
 
clarafok
Thanks Received: 5
Forum Guests
 
Posts: 98
Joined: December 27th, 2010
 
 
trophy
Most Thankful
trophy
First Responder
 

Q22 - In a study, one group

by clarafok Wed Feb 09, 2011 6:26 am

hello,

can someone please explain why the answer is A and not D? is it because we're trying to weaken the point that low-carb diet is the MOST effective way to lose body fat?

is A basically saying that high-carb diet could be the most effective way to lose body fat except that the water retained compensates for any weight lost? but it seems to me this doesn't really weaken the argument tho.

thanks in advance!
 
kgd91589
Thanks Received: 0
Forum Guests
 
Posts: 2
Joined: April 25th, 2011
 
 
 

Re: Q22 - In a study, one group

by kgd91589 Tue May 03, 2011 10:08 am

I could use some help with this problem as well. Can someone please provide an answer explanation please??

Thanks in advance.
 
ms.sylvialam
Thanks Received: 2
Forum Guests
 
Posts: 2
Joined: March 13th, 2011
 
This post thanked 2 times.
 
 

Re: Q22 - in a study, one group of volunteers

by ms.sylvialam Mon May 23, 2011 11:48 pm

I initially got this question wrong as well but I understand now knowing the answer is A.

It is important to note the most of the stimulus is speaking to WEIGHT. The stimulus talks about a study stating that those on LOW carb-HIGH protein diet lost MORE weight than the HIGH carb-LOW protein diet. However, the stimulus then concludes that the most effective way to lose body FAT (not weight) is to essentially follow the LOW carb-HIGH protein diet.

(A) is correct because if true, a HIGH carb-LOW protein diet may actual cause more loss in body FAT than the stimulus-sponsored LOW carb-HIGH protein diet (even though the LOW carb dieters lost more weight than the HIGH carb dieters). It says that most of the weight in a HIGH carb-LOW protein diet is caused by water retention compensating for the FAT loss.

(B) is wrong because it is out of the scope of the experiment. These people could be eating excess calories in protein and are not following any sort of diet whatsoever.

(C) is wrong because it doesn't weaken and maybe strengthens the stimulus, since the stimulus advocates for a LOW carb-HIGH protein diet. This answer would be right if the answer choice referred to a HIGH carb-LOW protein diet.

(D) is wrong, because it once again strengthens rather than weakens the argument. If the volunteers on the HIGH carb-LOW protein diet was not able to lose weight even despite the additional exercise, of course the stimulus-advocated LOW carb-HIGH protein diet would be more effective.

(E) is tricky. I feel if A wasn't there, this would be the correct answer. However, E does not address body fat at all, which is the essential part of the conclusion you are trying to weaken.

Hope this helps! Let me know if you have any additional questions. I think the trick of this question is to realize the insertion of body fat versus weight in the stimulus and keeping track of the low carb-high protein diet versus the high carb-low protein diet in the answer choices.
 
timmydoeslsat
Thanks Received: 887
Atticus Finch
Atticus Finch
 
Posts: 1136
Joined: June 20th, 2011
 
 
trophy
Most Thanked
trophy
First Responder
 

Re: Q22 - in a study, one group of volunteers

by timmydoeslsat Fri Aug 26, 2011 10:57 pm

I just did section 3 of this test as practice. This was the only question I missed. I believe I misread choice D as saying that the low carb group did the exercise and the high carb did not.

If this were indeed what it said, that would be the answer correct?

I did notice the term switch of the premises to the conclusion with body fat rather than weight loss.
 
farhadshekib
Thanks Received: 45
Elle Woods
Elle Woods
 
Posts: 99
Joined: May 05th, 2011
 
 
trophy
Most Thanked
 

Re: Q22 - in a study, one group of volunteers

by farhadshekib Wed Sep 07, 2011 3:35 pm

timmydoeslsat Wrote:I just did section 3 of this test as practice. This was the only question I missed. I believe I misread choice D as saying that the low carb group did the exercise and the high carb did not.

If this were indeed what it said, that would be the answer correct?

I did notice the term switch of the premises to the conclusion with body fat rather than weight loss.


Yes because this would mean that one group (i.e. low carb) was exposed to an extraneous factor - i.e. regular exercise - which could possibly account for the discrepancy in loss of body fat.

This was also the only question I missed on this section. I choose B, but I cannot exactly articulate why B is incorrect.

Is it because the conclusion suggests that the most effective way to lose body fat is to "eat much protein and shun carbs", while B only discusses the effects of increased protein intake?

So B is really out of scope in this context?
 
timmydoeslsat
Thanks Received: 887
Atticus Finch
Atticus Finch
 
Posts: 1136
Joined: June 20th, 2011
 
 
trophy
Most Thanked
trophy
First Responder
 

Re: Q22 - in a study, one group of volunteers

by timmydoeslsat Wed Sep 07, 2011 4:02 pm

I would also add that (B) does not mention the calories of those "many people." It is simply way too weak to do damage to this argument.

After having time away from this question, I do bring a different perspective to it now.

The issue in the argument is that we go from talking about 2 diets and the effects of it in terms of weight loss, and then we have a rather aggressive conclusion of the MOST effective way to lose BODY fat is to choose the higher weight loss one.

So we have a conclusion issue of swapping terms, going from body fat to weight loss...and we have an issue that was not tested in this question, but could have been, which is saying most effective way.

We know that one diet may be better than other in terms of weight loss, but even if it did say weight loss in the conclusion rather than body fact, we do not have enough evidence to say this is the most effective way.

Perhaps the most effective way is to eat low protein and low carbs?
User avatar
 
LSAT-Chang
Thanks Received: 38
Atticus Finch
Atticus Finch
 
Posts: 479
Joined: June 03rd, 2011
 
 
trophy
Most Thankful
trophy
First Responder
 

Re: Q22 - in a study, one group of volunteers

by LSAT-Chang Thu Oct 13, 2011 11:26 pm

I don't see how (B) is out of scope. Is (B) wrong because it doesn't address the "shunning carbohydrates" aspect? Could someone provide a clearer explanation as to why (B) is wrong? It was really tempting.
User avatar
 
ManhattanPrepLSAT1
Thanks Received: 1909
Atticus Finch
Atticus Finch
 
Posts: 2851
Joined: October 07th, 2009
 
 
 

Re: Q22 - in a study, one group of volunteers

by ManhattanPrepLSAT1 Fri Oct 21, 2011 7:34 pm

I totally see why answer choice (B) is tempting. It plays off the standard approach to weakening a common error on Weaken questions - correlation vs causation. In such a case providing the presumed cause without the presumed effect would undermine the argument.

This argument does commit the error of correlation vs causation, but it commits several other errors as well. Imagine the test-writer as a bull-fighter and he just waived his cape and you (the bull) just ran through his trap.

On harder questions there is an obvious gap in the reasoning and maybe several other less obvious gaps. An incorrect answer might point you in the direction of the obvious gap and thus look correct, but then slightly miss describing the error in the reasoning and thereby make itself not the correct answer.

Answer choice (B) misses because it's perfectly consistent with the argument's conclusion - which was about high-protein, low-carbohydrate diets. This answer choice is just about high-protein diets and whether that also includes high amounts of carbohydrates or not isn't conveyed.

Another gap in the reasoning here is the switch between losing weight and losing fat. This error is addressed in the correct answer - answer choice (A).

Hope that helps!
 
shirando21
Thanks Received: 16
Atticus Finch
Atticus Finch
 
Posts: 280
Joined: July 18th, 2012
 
 
 

Re: Q22 - In a study, one group

by shirando21 Wed Aug 08, 2012 12:03 pm

I still don't understand how A works to weaken the argument.

the conclusion in the argument is the most effective way to lose body fat is to eat much protein and shun carbohydrates.

according to A, a high-protein, low-carbohydrate diet does not cause the human body to retian water,the added weight of which largely compensates for the weight of any body fat lost.
To me, this still does not prove that a high-protein, low-carbohydrate diet causes more body fat loss.

Can anyone help to explain? Thanks.
 
shaynfernandez
Thanks Received: 5
Elle Woods
Elle Woods
 
Posts: 91
Joined: July 14th, 2011
 
This post thanked 1 time.
 
 

Re: Q22 - In a study, one group

by shaynfernandez Fri Aug 24, 2012 5:46 pm

The argument is based on a study where one group is given diet X and the other diet Y.
Diet Y lost more weight.
Therefore, the best way to lose fat is to use diet Y.

A. If nothing else attacks the equivocation of weight and fat by attacking the interpretation of the facts in the study. If A is added to the argument, diet X's weight is not representative of its ability to lose body fat. Why? Because diet X's measured weight was compromised by the increase in water. This attacks the assumption that the study is representative.
If your looking to destroy the argument this does not, but it does cast doubt on the strong statement that Y s the most effective way to lose body fat.
 
shirando21
Thanks Received: 16
Atticus Finch
Atticus Finch
 
Posts: 280
Joined: July 18th, 2012
 
 
 

Re: Q22 - In a study, one group

by shirando21 Thu Oct 25, 2012 6:11 pm

OK, so basically A is saying, although a low-protein, high-carbohydrate diet does not cause more weight loss, it actually causes more loss in body fat which challenges the reasoning in the argument: lose more weight--> lose more body fat by saying lose no more weight--> lose more body fat.
 
sch6les
Thanks Received: 5
Forum Guests
 
Posts: 13
Joined: July 24th, 2012
 
 
 

Re: Q22 - In a study, one group

by sch6les Fri Mar 08, 2013 3:59 pm

Low-carb diet correlated with lower weight than high-carb diet.
---
Therefore, low-carb diet cause lower body fat than high-carb diet.

There are two assumptions in this argument:
(1) Lower weight => lower body fat.
(2) Correlation => causation.

(A) supports the negation of (1). (A) states that the low-carb diet does not cause the body to at least maintain its water level. In other words, the low-carb diet causes the body to lower its water level.

So we have: lower weight => lower water level. This supports the idea that lower weight => ~lower body fat, i.e. the negation of (1). Therefore, (A) weakens the argument.
User avatar
 
ManhattanPrepLSAT1
Thanks Received: 1909
Atticus Finch
Atticus Finch
 
Posts: 2851
Joined: October 07th, 2009
 
 
 

Re: Q22 - In a study, one group

by ManhattanPrepLSAT1 Sun Mar 10, 2013 8:03 pm

shirando21 Wrote:OK, so basically A is saying, although a low-protein, high-carbohydrate diet does not cause more weight loss, it actually causes more loss in body fat which challenges the reasoning in the argument

I think maybe the exact opposite. Answer choice (A) says that even though a high-protein diet causes more weight loss, it does not cause more loss in body fat. The last part of which undermines the argument that a high-protein diet is the most effective way to lose body fat.
sch6les Wrote:Low-carb diet correlated with lower weight than high-carb diet.
---
Therefore, low-carb diet cause lower body fat than high-carb diet.

There are two assumptions in this argument:
(1) Lower weight => lower body fat.
(2) Correlation => causation.

Very nicely stated sch6les!

Incorrect Answers
(B) is irrelevant since these people could also be eating significant amounts of carbohydrates.
(C) is irrelevant since this only applies to what is often the case. Some isolated individuals cannot undermine what is true of the group at large.
(D) is a premise booster. This shows that had the low-carbohydrate group also exercised, they would have seen even more weight loss.
(E) is a premise booster. This supports the view that the low-carbohydrate diet did have the effect of losing weight. But where does that leave the argument's conclusion about body fat?
 
csunnerberg13
Thanks Received: 24
Elle Woods
Elle Woods
 
Posts: 62
Joined: April 10th, 2013
 
 
 

Re: Q22 - In a study, one group

by csunnerberg13 Sun Sep 29, 2013 10:04 am

Clarifying question: A is correct because it gives us a reason to think that lost weight actually tells us nothing about body fat, because the amount of weight lost relates differently to body fat in each of the 2 diets. A tells us that the supposedly worse diet gives us a weight that confounds the amount of body fat lost and it suggests that the better diet does not do this. However even so, we have not completely disproven the argument because it's possible that the better diet did in fact cause people to lose more body fat - it's just that knowing about weight will no longer allow us to make that conclusion because of (A).

Is this correct?
 
pewals13
Thanks Received: 15
Elle Woods
Elle Woods
 
Posts: 85
Joined: May 25th, 2013
 
This post thanked 1 time.
 
 

Re: Q22 - In a study, one group

by pewals13 Wed Aug 06, 2014 7:12 pm

The Mission:

To weaken the argument's conclusion that the most effective way to lose body fat is to eat much protein and shun carbohydrates

The Core:

In a study a group of volunteers who was on a high-protein, low carbohydrate diet lost more weight than a group of volunteers on a low protein, high carbohydrate diet
=>
The most effective way to lose body fat is to each much protein and shun carbohydrates

The Gaps:

First, just because this diet correlated with more weight loss in a particular group of people does not mean mean it is the most effective way to lose body fat

Second, their is a subtle term shift between weight loss in the premise and body fat loss in the conclusion

Answer Choices:

(A) CORRECT: This weakens the evidence provided for the conclusion- specifically that the loss of body weight by the group volunteers is equivalent to a loss of body fat. This answer tells us it is possible for body fat to be lost without weight loss due to water retention by those on high carb diets.

(B) Out of Scope: I fell for this one. It is WRONG because these people who consume large amounts of protein could also be consuming large amounts of carbohydrates. We don't know that they are actually on the low carb diet discussed in the stimulus.

(C) Out of Scope: Irrelevant, doesn't get at the core of the argument. The diet sometimes converts fat into muscle, so what?

(D) Opposite: If those on the high carbohydrate diet exercised and STILL lost less weight than the low carb diet volunteers who did not exercise, that low carb diet would appear to be pretty darn powerful

(E) Out of Scope: What happened to these folks off the diet is irrelevant to the argument
 
daijob
Thanks Received: 0
Elle Woods
Elle Woods
 
Posts: 74
Joined: June 02nd, 2015
 
 
 

Re: Q22 - In a study, one group

by daijob Sat Jul 25, 2015 12:40 pm

I'm still not sure why A is the answer...
I understand the first part of A implies that this group also decreased the amount of fat, but how the latter part undermines the conclusion? I thought it could be irrelevant.
I thought the group may or may not decrease fat amount. It is not sure from the statement because I thought it says "whereas a highprotein low-c diet does not causes the human body to retain water " and in that case they may lose weight but did not compensate for the loss of fat lost. so they actually lose the weight and fat.
(so the difference of weight is the amount of water)
The thing I was not sure in the conclusion is "the most effective way"...if A is true, the other group also lost fat, so it is actually not sure which was more effective to lose weight. But I guess this is not the reason A is correct reading above posts...
What did I misinterpret here??

Thank you
User avatar
 
tommywallach
Thanks Received: 468
Atticus Finch
Atticus Finch
 
Posts: 1041
Joined: August 11th, 2009
 
 
 

Re: Q22 - In a study, one group

by tommywallach Sun Jul 26, 2015 3:44 pm

There are 3-4 really good explanations of (A) in this thread already. I encourage you to read over them, as I don't think it can be explained any more clearly than it already has. Good luck!

-t
Tommy Wallach
Manhattan LSAT Instructor
twallach@manhattanprep.com
Image
 
daijob
Thanks Received: 0
Elle Woods
Elle Woods
 
Posts: 74
Joined: June 02nd, 2015
 
 
 

Re: Q22 - In a study, one group

by daijob Mon Jul 27, 2015 8:29 pm

Um, actually I read the explanations already and after that I asked this question...so I'm still not sure a little :|
User avatar
 
uhdang
Thanks Received: 25
Atticus Finch
Atticus Finch
 
Posts: 227
Joined: March 05th, 2015
 
 
 

Re: Q22 - In a study, one group

by uhdang Mon Aug 10, 2015 11:05 pm

By pointing out that the author falsely assumes weight change indicates fat change, A) weakens the author's reasoning of "gaining weight in case of high carb diet makes it less effective in losing fat than low carb diet", thereby weakening the conclusion.
"Fun"